Complex Number Question - Euler's Formula?

Click For Summary
The discussion focuses on expressing the complex number (-2 + 2i)^10 using Euler's formula. The correct approach involves raising both the modulus and the angle to the power of 10, resulting in r = (2√2)^10. The angle θ is calculated as 15π/2, which simplifies to 3π/2 when considering periodicity in polar coordinates. Participants clarify that angles can be reduced by subtracting multiples of 2π to achieve a more standard form. The final expression is confirmed as e^(i 3/2π), demonstrating a clear understanding of the application of Euler's formula.
KAISER91
Messages
27
Reaction score
0
Complex Number Question - Euler's Formula?

Homework Statement



Express (-2 + 2i)^10 in the form re^(iθ)

Homework Equations



Euler's Formula, I think?

The Answer given is: (2√2)^10 e^[i(15pi/2)]

The Attempt at a Solution

I don't know what I'm doing wrong here;

r = √[(-2)^2 + 2^2] = √8 = 2√2theta = pi - arctan (y/x)

= pi - arctan (2/-2)

= pi - 45

= pi - (pi/4)

= 3pi / 4Help will be appreciated. Thanks
 
Physics news on Phys.org


Well for one thing you didn't raise your r and theta to the power of 10.

(re^(ix))^10= r^10(e^(i(10x)). Your next step should be to reduce 10x to a smaller angle since angles repeat after 2pi.

Hope that helps.

If you need more explainatiom post again. :-):-)
 


╔(σ_σ)╝ said:
Well for one thing you didn't raise your r and theta to the power of 10.

(re^(ix))^10= r^10(e^(i(10x)). Your next step should be to reduce 10x to a smaller angle since angles repeat after 2pi.

Hope that helps.

If you need more explainatiom post again. :-):-)
Thanks for the reply mate.

So I have to raise r and theta to ^10.

That makes r = (2√2)^10

However how did theta become 15pi/2 ?
 


I think I got it.

Just bring down 10 as the power??

e^[10 x i(3pi/4)]
 


Yes and then simplify the angle. For example 7pi is the same as pi + 2(3)pi.

The answer you gave is correct but was not put in the "best" way.

So 10/4(3pi) = 15pi/2 = 7pi + pi/2 = 6pi + 3pi/2

Angles in polar form are usually written as i( theta + 2npi) :-)

We usually do not put the 2npi part since it is "implied" thus, your answer should be

e^(i 3/2(pi))
 


╔(σ_σ)╝ said:
Yes and then simplify the angle. For example 7pi is the same as pi + 2(3)pi.

The answer you gave is correct but was not put in the "best" way.

So 10/4(3pi) = 15pi/2 = 7pi + pi/2 = 6pi + 3pi/2

Angles in polar form are usually written as i( theta + 2npi) :-)

We usually do not put the 2npi part since it is "implied" thus, your answer should be

e^(i 3/2(pi))
OH!

I understand now. Thanks heeps for the help.
 
Question: A clock's minute hand has length 4 and its hour hand has length 3. What is the distance between the tips at the moment when it is increasing most rapidly?(Putnam Exam Question) Answer: Making assumption that both the hands moves at constant angular velocities, the answer is ## \sqrt{7} .## But don't you think this assumption is somewhat doubtful and wrong?

Similar threads

  • · Replies 22 ·
Replies
22
Views
966
Replies
1
Views
1K
  • · Replies 5 ·
Replies
5
Views
2K
  • · Replies 7 ·
Replies
7
Views
2K
  • · Replies 3 ·
Replies
3
Views
2K
  • · Replies 3 ·
Replies
3
Views
1K
  • · Replies 20 ·
Replies
20
Views
2K
  • · Replies 14 ·
Replies
14
Views
2K
  • · Replies 4 ·
Replies
4
Views
2K